www.vorhilfe.de
- Förderverein -
Der Förderverein.

Gemeinnütziger Verein zur Finanzierung des Projekts Vorhilfe.de.
Hallo Gast!einloggen | registrieren ]
Startseite · Mitglieder · Impressum
Forenbaum
^ Forenbaum
Status VH e.V.
  Status Vereinsforum

Gezeigt werden alle Foren bis zur Tiefe 2

Navigation
 Startseite...
 Suchen
 Impressum
Das Projekt
Server und Internetanbindung werden durch Spenden finanziert.
Organisiert wird das Projekt von unserem Koordinatorenteam.
Hunderte Mitglieder helfen ehrenamtlich in unseren moderierten Foren.
Anbieter der Seite ist der gemeinnützige Verein "Vorhilfe.de e.V.".
Partnerseiten
Weitere Fächer:

Open Source FunktionenplotterFunkyPlot: Kostenloser und quelloffener Funktionenplotter für Linux und andere Betriebssysteme
Forum "Uni-Analysis" - Beweis von Integrationsregel
Beweis von Integrationsregel < Analysis < Hochschule < Mathe < Vorhilfe
Ansicht: [ geschachtelt ] | ^ Forum "Uni-Analysis"  | ^^ Alle Foren  | ^ Forenbaum  | Materialien

Beweis von Integrationsregel: Aufgabe
Status: (Frage) reagiert/warte auf Reaktion Status 
Datum: 19:08 Fr 23.06.2006
Autor: shark4

Aufgabe
Beweisen Sie die Beziehung
[mm]\int x^{n} \sin x \, d x = - \sum_{k = 0}^{n} k! {n \choose k} x^{n - k} \cos(x + k \pi /2) + c[/mm]
und stellen Sie eine analoge Formel für [mm]\int x^{n} \cos x \, d x[/mm] auf!

Ich weiß zwar das es stimmt, aber ich weiß nicht wie ich es beweisen soll.

Hier mein Ansatz:
Die eine Möglichkeit wäre über Induktion:
Für n = 1 und n = 2 kann man es schnell nachrechnen.
Vorausetzung: Formel gilt bis n
Schritt:
[mm]\begin{matrix} \int x^{n+2}\sin x\, dx &=& -x^{n+2} \cos x+(n+2)\int x^{n+1} \cos x\, dx \\ &=& -x^{n+2}\cos x+(n+2)\left(x^{n+1}\sin x-(n+1)\int x^{n} \sin x\, dx\right) \\ &=& -x^{n+2}\cos x+(n+2)x^{n+1}\sin x-(n+2)(n+1)\left(- \sum_{k=0}^{n} k! {n \choose k} x^{n-k} \cos(x+k \pi /2)+c\right)\\ &=& -\sum_{k=0}^{2} k! {n+2 \choose k} x^{n+2-k} \cos(x+k \pi /2)-\left(- \sum_{k=0}^{n} (n+2)(n+1)k! {n \choose k} x^{n-k} \cos(x+k \pi /2)+c\right) \end{matrix}[/mm]
Jetzt muss man nur "noch" die rechte Summe in die Form
[mm]\sum_{k=2}^{n+2} k! {{n+2} \choose k} x^{n+2-k} \cos(x+k \pi /2)[/mm]
bringen. Aber wie?
[mm]\begin{matrix} - \sum_{k=0}^{n} (n+2)(n+1)k! {n \choose k} x^{n-k} \cos(x+k \pi /2)+c &=& \sum_{k=0}^{n} (n+2)(n+1)k! {n \choose k} x^{n-k} \left(-\cos(x+k \pi /2)\right)+c \\ &=& \sum_{k=0}^{n} (n+2)(n+1)k! {n \choose k} x^{n-k} \cos(x+(k+2) \pi /2)+c \\ &=& \sum_{k=0}^{n} k! {{n+2} \choose {k+2}} x^{n-k} \cos(x+(k+2) \pi /2)+c \end{matrix}[/mm]
Bis hierher komm ich noch, aber wie gehts weiter?

Die zweite Möglichkeit besteht darin viermal die partielle Integration anzuwenden:
[mm]\begin{matrix} \int x^{n}\sin x\, dx &=& -x^{n} \cos x-(n)\int -x^{n-1} \cos x\, dx \\ &=& -x^{n}\cos x-(n)\left(-x^{n-1}\sin x-(n-1)\int -x^{n-2} \sin x\, dx\right) \\ &=& -x^{n}\cos x-(n)\left(-x^{n-1}\sin x-(n-1)\left(x^{n-2} \cos x - (n-2)\int x^{n-3} \cos x\, dx\right)\right) \\ &=& -x^{n}\cos x-(n)\left(-x^{n-1}\sin x-(n-1)\left(x^{n-2} \cos x - (n-2)\left(x^{n-3} \sin x - (n-3)\int x^{n-4} \sin x\, dx\right)\right)\right) \\ &=& -x^{n}\cos x-(n)\left(-x^{n-1}\sin x-(n-1)\left(x^{n-2} \cos x - (n-2)x^{n-3} \sin x +(n-2)(n-3)\int x^{n-4} \sin x\, dx\right)\right) \\ &=& -x^{n}\cos x-(n)\left(-x^{n-1}\sin x-(n-1)x^{n-2} \cos x + (n-1)(n-2)x^{n-3} \sin x-(n-1)(n-2)(n-3)\int x^{n-4} \sin x\, dx\right) \\ &=& -x^{n}\cos x+(n)x^{n-1}\sin x+(n)(n-1)x^{n-2} \cos x-(n)(n-1)(n-2)x^{n-3} \sin x+(n)(n-1)(n-2)(n-3)\int x^{n-4} \sin x\, dx \\ &=& -x^{n}\cos x-(n)x^{n-1}\cos (x+\pi /2)-(n)(n-1)x^{n-2} \cos (x+\pi)-(n)(n-1)(n-2)x^{n-3} \cos (x+3/2\pi)+(n)(n-1)(n-2)(n-3)\int x^{n-4} \sin x\, dx \\ \end{matrix}[/mm]
Ab [mm](n)(n-1)(n-2)(n-3)\int x^{n-4} \sin x\, dx[/mm] wiederholt sich das ganze. Also müsste man dort mit der Periodizität argumentieren können. Oder reicht das nicht?

Ich habe diese Frage in keinem Forum auf anderen Internetseiten gestellt.

        
Bezug
Beweis von Integrationsregel: Antwort
Status: (Antwort) fertig Status 
Datum: 19:30 Fr 23.06.2006
Autor: Walde

Hi shark4,

uff, die Formeln erschlagen mich etwas, also kontrolliere ich sie mal nicht nach.
Aber folgendes: Per Induktion geht es wahrscheinlich auch, aber deine zweite Überlegung, viermal part. zu intergrieren und dann das Periodizitätsargument zu bringen ist meiner Meinung nach genau das Richtige. Man kann es ja eigentlich schon sehen. Wenn du von dem 4maligen p. int. auf n-maliges geschlossen hast bzw. mal die ersten 4 Glieder der Summenfomel ausgeschrieben hast, steht es ja quasi da.

L G walde

Bezug
Ansicht: [ geschachtelt ] | ^ Forum "Uni-Analysis"  | ^^ Alle Foren  | ^ Forenbaum  | Materialien


^ Seitenanfang ^
ev.vorhilfe.de
[ Startseite | Mitglieder | Impressum ]